Practise Questions FRCR CO2A Flashcards

1
Q
  1. A 20-year-old man presents with a metastatic testicular tumour. Which of the following features would put him into the poor-risk category?
    (a) Pure seminoma with 20 lung metastases
    (b) Pure seminoma with hCG 50× upper limit of
    normal (ULN)
    (c) Choriocarcinomawith20lungmetastases
    (d) Embryonal germ cell tumour (GCT) with
    αFP 50× ULN
    (e) MixedGCTwith10lungand2liver
    metastases
A

(e) Mixed GCT with 10 lungand2liver
metastases

How well did you know this?
1
Not at all
2
3
4
5
Perfectly
2
Q

With regard to cancer-related emergencies, which of the following statements is correct?
(a) Syndromeofinappropriateanti-diuretic hormone (SIADH) gives rise to hyponatraemia and reduced plasma osmolarity in the presence of inappropriately dilute urine
(b) In patients with SIADH who do not respond to fluid restriction, cautious administration of intravenous sodium chloride should be considered
(c) Theadditionofsurgerytoradiotherapyhas been shown to improve functional outcome compared to surgery alone in the treatment of spinal cord compression
(d) In good-performance-status patients with solitary intracerebral metastases, radiotherapy with 12 Gy in 2 fractions is equivalent to 30 Gy in 10 fractions
(e) Patientsathighriskfromneutropenicsepsis should receive intravenous ciprofloxacin and co-amoxiclav on admission to hospital

A

c) Theadditionofsurgerytoradiotherapyhas been shown to improve functional outcome compared to surgery alone in the treatment of spinal cord compression

How well did you know this?
1
Not at all
2
3
4
5
Perfectly
3
Q

A 45-year-old man presents with an enlarging mass on the penis. Biopsy reveals invasive squamous carcinoma. The tumour is invading the corpus cavernosum and there are two mobile, palpable inguinal node metastases. What is
the stage?
(a) T3 N1 MX
(b) T1 N1 MX
(c) T3 N2 MX
(d) T2 N1 MX
(e) T2 N2 MX

A

(e) T2 N2 MX

How well did you know this?
1
Not at all
2
3
4
5
Perfectly
4
Q

Xeroderma pigmentosa is associated with which of the following?
(a) Autosomaldominantinheritance
(b) Jaw cysts
(c) Medulloblastoma
(d) A defect of nucleotide excision repair (e) Adecreasedriskofskincancer

A
How well did you know this?
1
Not at all
2
3
4
5
Perfectly
5
Q

A patient presents with multiple bilateral pulmonary opacities suggestive of malignancy but with no obvious underlying primary site of disease. A CT-guided biopsy reveals adenocarcinoma on which the following immunohistochemical tests are performed: cytokeratin-7 (CK7) positive, cytokeratin-20 (CK20) negative, thyroid transcription factor (TTF-1) positive, thyroglobulin negative. The most likely underlying primary site of disease is which of these?
(a) Colon (b) Bladder (c) Lung
(d) Thyroid (e) Pancreas

A
How well did you know this?
1
Not at all
2
3
4
5
Perfectly
6
Q

A 53-year-old woman presents with a painless right-sided thyroid nodule and normal TFTs. There is no family history of thyroid disease or endocrine malignancy. US confirms a solitary 3 × 2 cm hypoechoic nodule with no abnormal nodes. FNA is reported as showing a follicular neoplasm. The next most appropriate step is:
(a) Serumthyroglobulinmeasurement
(b) Repeat FNAC
(c) TorequestCTscanoftheneck,thoraxand
abdomen with intravenous contrast
551
(d) To advise a right-sided thyroid lobectomy
(e) Total thyroidectomy and central lymph node
dissection

A
How well did you know this?
1
Not at all
2
3
4
5
Perfectly
7
Q

A 54-year-old man presents with a 6-month history of a hoarse voice and no other ENT symptoms. Clinic room examination reveals
a proliferative lesion on the left false cord
with normal true cord mobility and palpable lymphadenopathy in level II on the ipsilateral side. EUA and CT scan confirm involvement of the false cord with extension along the aryepiglottic foldtowardstheepiglottisandinvolvementofthe pre-epiglottic space. There is a left level II lymph node measuring 3 cm. There is no evidence of distant metastases. Biopsy from the false cord shows moderately differentiated squamous carcinoma. According to the 7th edition TNM classification, what stage disease is this?
(a) T2 N1 M0
(b) T2 N2a M0
(c) T2 N2b M0
(d) T3 N1 M0
(e) T3 N2a M0

A
How well did you know this?
1
Not at all
2
3
4
5
Perfectly
8
Q

A 54-year-old patient has hepatocellular carcinomaassociatedwithhepatitisCinfection. Investigations reveal three tumours in the right lobe of the liver: 2 cm, 2 cm and 3 cm in size. There is no evidence of metastatic disease. Which of the following options is the treatment of choice?
(a) Radiofrequencyablation
(b) Liver transplantation
(c) Transarterialchemoembolisation
(d) External beam radiotherapy
(e) Noneoftheaboveissuperiortotheothers

A
How well did you know this?
1
Not at all
2
3
4
5
Perfectly
9
Q

In electron therapy, which of the following statements is correct?
(a) The therapeutic range (on the central axis) is approximately E/2 cm (where E is the energy of the incident electron beam in MeV)
(b) Unlike kV photons, electron treatments rarely require the use of bolus
(c) The 80% isodose is wider at depth than at the surface
(d) Hot and cold spots caused by inhomogeneities can be easily corrected for, provided their position is known
e) The10%isodoseiswideratdepththanatthe surface

A
How well did you know this?
1
Not at all
2
3
4
5
Perfectly
10
Q

Whichofthefollowingconditionsismost typically associated with an increase in colorectal adenocarcinomas?
(a) Neurofibromatosis
(b) Xeroderma pigmentosa
(c) AgermlinemutationinBRCA1
(d) Turcot’s syndrome
(e) AgeneticdefectintheHedgehogpathway

A
How well did you know this?
1
Not at all
2
3
4
5
Perfectly
11
Q

A54-year-oldwomanpresentswithanenlarging mass on the vulva that has been present for
12 months. A biopsy confirms this is squamous carcinoma. There is no other finding on examination. The tumour involves the right
side of the vulva, measures 8 cm in maximum diameter and extends to within 3 mm of the anal margin. She does not wish to have an anovulvectomy. What is the most appropriate treatment?
(a) Localexcision
(b) Radical vulvectomy and groin node
dissection
(c) Radical radiotherapy to include the primary
tumour with an electron field to the
perineum
(d) Radical chemoradiotherapy to the vulva and
groin nodes
(e) Symptomatic treatment

A
How well did you know this?
1
Not at all
2
3
4
5
Perfectly
11
Q

A65-year-oldmanwithpancreaticcarcinoma undergoes a Whipple’s operation. Final histology shows a T3 N1 (5 of 15 nodes positive) grade 3 adenocarcinoma with vascular invasion, distance to
closest resection margin is 1 mm. Based on current evidence (and outside the context of a clinical trial), what is the most appropriate management?
(a) Noadjuvanttherapy
(b) Adjuvant 5-FU chemotherapy
(c) Adjuvantgemcitabinechemotherapy (d) Adjuvant chemoradiation
(e) Re-operation

A
How well did you know this?
1
Not at all
2
3
4
5
Perfectly
12
Q

A62-year-oldwomanpresentswith postmenopausal bleeding. A hysteroscopy shows a tumour in the uterus which on biopsy is found to be a well-differentiated adenocarcinoma. An MRI scan suggests that the tumour is invading
3 mm into the myometrium, with a myometrial thickness of 7 mm. What initial treatment would you recommend?
(a) TAH, BSO and pelvic lymphadenectomy (b) TAH, BSO
(c) Radical radiotherapy
(d) Intrauterine progesterone
(e) TAH,BSOandinfracolicomentectomy

A
How well did you know this?
1
Not at all
2
3
4
5
Perfectly
13
Q

ApatientpresentingwithSVCOisgivenoxygen, commencedonhigh-dosesteroidsandhis conditionstabilises.CTscanningconfirmsthe presence of a large superior mediastinal mass. What is the most appropriate management at this stage?
(a) Radiotherapytothemediastinumusing parallel opposed anterior and posterior fields; dose 20 Gy in 5 fractions over 5 days
(b) Chemotherapy with BEP
(c) Referralforbiopsy
(d) Anticoagulation
(e) Insertionofapercutaneousstentinto
the IVC

A
How well did you know this?
1
Not at all
2
3
4
5
Perfectly
14
Q

ForsomeoneonMST60mgb.d.,whatshouldthe breakthrough dose of oral morphine be?
(a) 20 mg
(b) 10 mg
(c) 30 mg
(d) 40 mg
(e) 60 mg

A
How well did you know this?
1
Not at all
2
3
4
5
Perfectly
14
Q

Anotherwisefitandwell61-year-oldman presents with dysphagia and is found to have
an adenocarcinoma of the oesophagus. A CT scan shows no evidence of metastatic disease.
An EUS confirms the tumour to be from 36 to
39 cm ab oral, extending into the right crus of the diaphragm together with two malignant lymph nodes in the peritumoural region. What stage
is this and what is the standard treatment in the UK?
(a) StageT3N1M0:4cyclesepirubicin,cisplatin and capecitabine (ECX) chemotherapy prior to surgery
(b) Stage T3 N1 M1b: definitive chemoradiotherapy
(c) StageT4N1M0:2cyclescisplatinand5-FU followed by surgery
19.
(d) Stage T4 N1 M1b: 2 cycles cisplatin and 5-FU followed by surgery
(e) Stage T3 N1 M0: 2 cycles cisplatin and capecitabine followed by surgery

A
How well did you know this?
1
Not at all
2
3
4
5
Perfectly
15
Q

Theage-specificincidenceinEnglandandWales is decreasing for which of these cancers?
(a) Pancreaticcancer
(b) Gastric cancer
(c) Colorectalcancer
(d) Oesophagealcancer
(e) Testiculargermcelltumours

16
Q

Criteriaforurgentreferralwithsuspicionof colorectal cancer include:
(a) Rectalbleedingwithanalsymptomsina 62-year-old
(b) Eight weeks change in bowel habit without rectal bleeding in a 45-year-old
(c) Persistentrectalbleedingwithoutanal symptoms in a 35-year-old
(d) Unexplained iron deficiency anaemia in men Hb < 110 g/L
(e) Leftiliacfossamass

17
Q

Apreviouslyfit46-year-oldwomanisfoundto have a 4 cm basaloid carcinoma of the anal canal. On examination she has a palpable lymph node in her left groin. There is no evidence of distant metastatic disease. An FNA of the groin node
is negative for malignancy. What is the correct initial treatment?
(a) Surgeryalonebecausebasaloidtumoursare not radiosensitive
(b) Cisplatin/5-FU concurrent with radiotherapy to the GTV plus 3 cm
(c) 5-FU plus mitomycin concurrent with radiotherapy to the GTV plus 3 cm
(d) 5-FU and mitomycin concurrent with radiotherapy to the stage-defined CTV
(e) Cisplatin and 5-FU concurrent with radiotherapy to the stage-defined CTV

18
Q

Regardingradicalradiotherapyforprostate cancer, which of the following statements is correct?
(a) TheGTVtoPTVmarginisusually2cm
(b) 3D conformal radiotherapy allows dose
escalation to 64 Gy
(c) Ifincludedinthetreatment,anappropriate seminal vesicle dose is 56 Gy
(d) The use of neoadjuvant hormone therapy causes greater rectal toxicity
(e) A three-field plan with an anterior field and two laterals causes greater rectal toxicity than a three-field plan with an anterior field and two posterior obliques

19
Q
  1. An otherwise fit 45-year-old woman presents with epigastric discomfort. An endoscopy
    reveals a smooth 3 cm submucosal lesion at the gastric cardia. A CT scan is consistent with a gastrointestinal stromal tumour. What would you recommend next?
    (a) Widelocalsurgicalresection
    (b) Endoscopic ultrasound-guided FNA (c) Imatinib
    (d) A D2 total gastrectomy
    (e) APETscan
20
Q

TheMRCCR07studyhasdemonstrated:
(a) Animprovementinoverallsurvivalwith preoperative radiotherapy
(b) A reduction in CRM positivity with pre- over postoperative radiotherapy
(c) Asignificantreductioninlocalrecurrence confined to lower-third tumours in patients receiving preoperative radiotherapy
(d) A significant reduction in local recurrence in patients receiving selective postoperative chemoradiotherapy
(e) Asignificantreductioninlocalrecurrencein patients receiving preoperative radiotherapy

21
Q

An84-year-oldwomanpresentstoan ophthalmologist with diplopia and reduced
visual acuity in her left eye. A CT scan shows a left retro-orbital mass. She had a mastectomy 10 years previously for a left breast cancer, followed by tamoxifen for 5 years. Other staging scans are negative. The CA15-3 is 310 u/mL. The original tumour was ER+ve. The most appropriate management now is:
(a) Anastrozole
(b) Urgent palliative radiotherapy to orbit
(c) Urgentpalliativeradiotherapytoorbitand
anastrazole
(d) Palliative chemotherapy
(e) Surgicalexcisionofthemass

22
Q

Whichofthefollowingstatementsaboutopioids is correct?
(a) Morphineisglucuronatedintheliver,so renal impairment does not lead to opioid toxicity
(b) Tachycardia is a characteristic feature of opioid toxicity
(c) Patientswithopioidtoxicitymaythinkthey have seen animals running under their bed
(d) Low-dose morphine is classed as a ‘weak opioid’ on the WHO analgesic ladder
(e) Opioidtoxicitytypicallycausespupil
dilatation

23
Q

Afit60-year-oldwomanundergoessurgeryfor a carcinoma of the tranverse colon. Histology shows a grade 2 adenocarcinoma, T3 N2 (4 out of 14 nodes involved) with vascular invasion and perineural invasion. There is no evidence of metastatic disease. What is the most appropriate management?
(a) Weekly5-FU/folinicacid
(b) Modified de Gramont 5-FU
(c) Oxaliplatin–fluoropyrimidine-based
regimen
(d) Oxaliplatin–capecitabine
(e) Irinotecan–modified de Gramont

24
Q

A 45-year-old man presents with gradual onset of right-sided weakness. An MRI scan of the brain shows an enhancing tumour in the right frontal and parietal region. The tumour is excised surgically and histology shows a glioma with frequent mitoses and necrosis. It is decided to treat with radical postoperative radiotherapy. Which of the following statements is correct?
(a) Treatingeachfieldeachday,Mondayto Friday, 5 days a week, the dose to the ICRU reference point should be 54 Gy in 30 fractions
(b) Craniospinal axis radiotherapy improves survival by reducing the risk of recurrence elsewhere in the central nervous system
(c) CTandMRIimageco-registrationisnot usually helpful in planning because of difficulties correcting for scale, rotation and lateral translation
(d) The GTV is the contrast-enhancing tumour as seen on imaging
e)If megavoltage photons are used, alopecia is unlikely because of the skin sparing in the build-up region

25
Q

Whichofthefollowingisassociatedwithapoor prognosis in neuroendocrine tumours?
(a) Well-differentiatedtumour (b) High Ki-67 index
(c) Originintheappendix
(d) Bronchial origin
(e) Absenceofatypicalcells

26
Q

Whichofthefollowingstatementsabout radiotherapy for thyroid eye disease is correct?
(a) Itshouldbegivenprophylactically
(b) Proptosis usually pushes the lenses so far
forward that their radiation dose can be
ignored
(c) NICErecommendsavoidingradiotherapy
because of the risk of long-term side effects
(d) The long-term risk of cataract is 10%
(e) The usual dose is 40 Gy in 20 fractions over
4 weeks

27
Q

Whichofthefollowingstatementsregarding retinoblastoma is correct?
(a) 40%ofcasesareassociatedwithagermline loss of one of the RB1 alleles
(b) External beam radiotherapy is the treatment of choice
(c) Chemotherapyshouldbeavoidedbecauseof the risk of significant toxicity
(d) Enucleation is the treatment of choice
(e) Smalltumoursdonotrequireimmediate
treatment and can be observed

28
Q

A43-year-oldmanpresentswithaswellingin the right thigh. Investigations show a soft tissue mass within the quadriceps muscle. A biopsy
is undertaken, followed by surgery to remove the tumour. Histology shows a leiomyosarcoma with 20 mitoses per high-power field and more than 50% necrosis. The tumour measured 3 cm diameter and had an excision margin of 25 mm. According to the TNM 7th edition classification, what stage is the tumour?
(a) pT1 NX MX
(b) pT2 N0 M0
(c) pT1 N0 M0
(d) R1 NX MX
(e) G3 T2 N0 M0

29
Q

A63-year-oldmaleisbeingplannedfor potentially curative definitive platinum-based chemoradiationforasquamouscellcarcinoma starting at 30 cm ab oral using a two-phase technique: phase 1 with anterior–posterior parallel-opposed fields; phase 2 with three fields, one anterior and two posterior
obliques. You are asked to review the
plans: PTVmax = 106%; PTVmin = 95%; the spinal cord V45 = 20%; combined lung V20 = 10%; and heart V30 = 70%. What action do you recommend?
(a) Acceptthecurrentplan
(b) Increase the dose contribution from
phase 2
(c) Increase the contribution of the anterior
field during phase 1
(d) Increase the dose contribution from phase 1 (e) Increase the contribution of the posterior
field during phase 1

30
Q

A45-year-oldmathsteacherhasjustcompleted adjuvant treatment for a right breast cancer. She had a 2.0 cm grade 3 tumour, with 1 of 17 nodes involved. The tumour was ER +ve but HER-2 − ve. She has had 6 cycles of FEC chemotherapy and radiotherapy to the conserved breast. She has started tamoxifen. She wants to know what her prognosis is. According to the Nottingham prognostic index, what is her 10-year survival likely to be?
(a) 51% (b) 41% (c) 61% (d) 71% (e) 31%

31
Q

A57-year-oldfitwomanpresentswithabdominal distension, poor appetite and weight loss. CT scan shows bilateral ovarian masses and an omental ‘cake’. Her CA125 is 763 U/mL. She undergoes laparotomy and debulking. Tumour deposits are found in the ovaries and on the omentum and surface of the liver measuring 2.5 cm. What stage is her ovarian cancer?
(a) FIGOstageIIIC (b) FIGO stage IVA (c) FIGOstageIIC
(d) FIGO stage IIIB (e) FIGOstageIIIA

31
Q

Whichofthefollowingstatementsabout radiotherapy for vulval cancer is correct?
(a) Thedoseofconcurrentweeklycisplatinis 50 mg/m2
(b) Moist desquamation of the vulval skin can occur, but it is not usually painful
(c) Thevulvadoesnottolerateradiotherapy well, so radiation doses in excess of 60 Gy cannot be given
(d) The clinical target volume in advanced vulval cancer includes the primary tumour, vulva, groin and pelvic nodes
(e) Postoperativeradiotherapydoesnot reduce the local recurrence rate in patients with a resection margin of less than 8 mm

31
Q

Inmegavoltagephotontherapy,whichofthe following statements is correct?
38.
(a) The width of the radiation beam increases linearly with distance from the treatment head because of the inverse-square law
(b) The beam penumbra is the distance between the 30% and 70% depth doses
(c) As the energy increases, the dmax decreases (d) When the distance from the treatment
machine to the patient increases, the inverse-square law causes percentage depth doses below the dmax to increase
(e) Asthefieldsizeincreases,thecentralaxis receives less radiation per monitor unit because of reduced scatter from the machine head and within the patient

32
Q

Whichofthefollowingstatementsabout radiotherapy for mesothelioma is correct?
(a) Wide-fieldpalliativeradiotherapycan alleviate pain in around 60% of patients
(b) Palliative radiotherapy to the chest for mesothelioma usually alleviates shortness of breath
(c) Thedoseforprophylacticneedle-track radiotherapy can be kept as low as 50 Gy in 25 fractions because only microscopic disease is being treated
(d) In postoperative radiotherapy following extrapleural pneumonectomy, the GTV is the entire hemithorax
(e) Symptomatic disease in the thorax should be treated with high-dose palliation

32
Q

Whichoftheseisthecorrectestimated5-year overall survival following an Ivor–Lewis oesophagectomy on a fit patient with a T3 N1 adenocarcinoma?
(a) 60% (b) 40% (c) 5% (d) 20% (e) 0%

33
Q

Whichofthefollowingaspectsofreportingtrials can actually lead to less reliable conclusions?
(a) Ensuring as complete a follow-up as possible
(b) Reporting trials whether or not they show a ‘positive’ or ‘negative’ result
(c) Stressingtoreadersthoseoutcomeswhere there is a difference between subgroups
(d) Performing a meta-analysis
(e) Reporting treatment effects and confidence
intervals

33
Q

Whichoneofthefollowingcytotoxicdrugsisa vesicant when extravasated – that is, is associated with a high risk of causing inflammation and blistering of local skin and underlying tissue leading to tissue death and necrosis?
(a) Carboplatin (b) Bleomycin (c) Paclitaxel
(d) Etoposide
(e) Methotrexate

34
Q

Whichofthefollowingstatementsrelatingtothe cervical lymph nodes is correct?
(a) LevelIIcontainsthesubmandibularnodes
(b) Preauricular and intraparotid nodes are
included in level I
(c) Lower jugular nodes lie in level III
(d) Level III extends from the hyoid bone
superiorly to the lower border of the cricoid
cartilage
(e) LevelVIcontainstheanteriorcentral
compartment nodes and extends between
39.
40.
the hyoid bone superiorly and the carina inferiorly